4
$\begingroup$

For a positive integer $m$, let $\mathcal{A}(m)$ be the set of all integers $k \geq 5$ such that: there is a positive integer $n$ and a subgroup $G \subset \operatorname{GL}_m(\mathbb{Z}/n\mathbb{Z})$ such that the alternating group $A_k$ is a composition factor of $G$ (i.e., writing $G$ as an iterated extension of finite simple groups, at least one of them is isomorphic to $A_k$).

Is $\mathcal{A}(m)$ is finite for all $m \in \mathbb{Z}^+$?

I have asked several questions here recently of the form "It would be great if $X$ were true. It seems unlikely, but I might as well ask." This time it would be nice if this were true, and I would be quite surprised if it were false.

(Some motivation: let $K$ be a field, and let $A_{/K}$ be an abelian variety. Suppose we want to build nontorsion points on $A(\overline{K})$. This cannot be done in general -- certainly it cannot be done when $K$ is algebraic over a finite field. But suppose that $K$ is a Hilbertian field, so for all $k \in \mathbb{Z}^+$ there is an $A_k$-Galois extension $L_k/K$. Then an affirmative answer to the above shows that $K(A(\overline{K})[\operatorname{tors}])$ contains only finitely many of the fields $L_k/K$. There will be points in $A(\overline{K})$ whose field of definition contains $L_k$, and these points cannot be torsion points. In fact if this is true then one can deduce the precise structure of $A(\overline{K})$ for any field $K$.)

There was a previous question on this site that was less ambitious but still interesting and relevant: it asked for examples of groups which cannot be subgroups of $\operatorname{GL}_2(\mathbb{Z}/p\mathbb{Z})$ for a prime $p$. The accepted answer used the structure of Sylow $p$-subgroups, which was something I hadn't thought of. It is plausible to me that one might be able to answer the question by showing that for all $m$, if $k$ is large enough then for all $n$, the $2$-Sylow subgroups ($2$ here is playing the role of a prime which is much smaller than $k$) of $\operatorname{GL}_m(\mathbb{Z}/n\mathbb{Z})$ have nilpotency class smaller than the nilpotency class of the $2$-Sylow subgroups of $A_k$. This would be sufficient, I believe.

If the answer turns out to be false, then I would be interested in hearing if you can make it true by replacing $A_k$ by a different infinite set of finite simple groups, especially a set each element of which is known to occur as a Galois group over every Hilbertian field.

Added: Finally an affirmative answer: hooray. What I really didn't know was the magnificent Larsen-Pink Theorem. As Peter Mueller says, that really makes things easy. Still, it makes it look like the result requires 21st century technology, and I strongly suspect that that is not the case. Although the proof I wrote below is the one I'll use in my paper, I am still interested to see other, more elementary proofs. (And I wonder if the idea of using nilpotency classes actually works...)

$\endgroup$

2 Answers 2

5
$\begingroup$

Latest Edit: Below I put a sketch on how to replace the deep Larsen-Pink Theorem by easier and more familiar results.

Edit: There was a silly mistake in showing that we may assume that $n$ is a prime. Still, it is true, see Pete L. Clark's fix in his answer, which also enhances my rather sketchy answer. Here is the rest of my original answer (with $p=n$):

Next, by passing to a smaller group in the composition series of $G$, you may assume that $A_k$ is a homomorphic image of $G$. However, that bounds $k$ in terms of $m$. There should be an easy argument. A sledgehammer argument uses a positive characteristic analogue of Jordan's upper bounds on linear groups by Larsen and Pink: There is a constant $j$ depending only on $m$, not on $p$, such that $G$ has normal subgroups $M\le N\le G$ where $[G:N]\le j$, $N/M$ is a direct product of simple Lie type groups of characteristic $p$, and $N$ is solvable.

Avoiding Larsen-Pink: Let $G/N$ be isomorphic to $A_k$, and suppose that $G\le GL_m(p)$. We need to show that $k$ is bounded in terms of $m$. Given $k,m$ and $p$, suppose that $\lvert G\rvert$ is minimal. Then each maximal subgroup $H$ of $G$ is contained in $N$, for otherwise $G=HN$, hence $H/(H\cap N)=A_k$, and we could replace $G$ by $H$. Thus $N$ is contained in the intersection $\Phi(G)$ of all maximal subgroups of $G$. Note that $\Phi(G)$ is called the Frattini subgroup of $G$, and it is well known (and easy to prove) that $\Phi(G)$ is nilpotent. In particular $N$ is nilpotent. (Actually $N=\Phi(G)$.) Let $s$ be a prime divisor of $\lvert N\rvert$, and $S$ be a Sylow $s$-subgroup of $N$. Note that all Sylow subgroups of $N$ are normal in $N$, and so they are normal in $G$ too. If $s$ is bigger than $k$, then $S$ is a normal Sylow subgroup of $G$. But then $G=S\rtimes H$ for a subgroup $H$ of $G$ by Schur-Zassenhaus, contrary to $HN\lt G$. So each prime divisor of $\lvert G\rvert$ is $\le k$. On the other hand, as $G\le GL_m(p)$, we have $k!\le p^{m^2}$. For fixed $m$, and $k$ big enough, this forces $p>k$, so $p$ does not divide the order of $G$. A standard result shows that $G$ can be lifted to characteristic $0$, that means there is a field $K$ of characteristic $0$ with $G\le GL_m(K)$. See e.g. Corollary 3.8 in Dixon's The structure of linear groups. Finally we can apply the old theorem of Jordan: There is a function $J$ on the positive integers such that if $G\le GL_m(K)$, then $G$ has an abelian normal subgroup $A$ with $\lvert G/A\rvert\le J(m)$, and the result follows.

$\endgroup$
1
  • 1
    $\begingroup$ (I deleted some comments after thinking things through.) I believe what you say in your second sentence is literally true if $n$ is a power of $p$ and generally not otherwise. (So it's true with $\mathbb{Z}$ replaced by $\mathbb{Z}_p$.) But the general idea looks very good. Thanks a lot, and I'll report back after I work it out. $\endgroup$ Jul 27, 2013 at 0:20
3
$\begingroup$

Here is my attempt at fleshing out Peter Mueller's answer. If anyone sees any inaccuracies (or worse) here, I would be grateful if you could let me know. If this answer seems correct to you, please be sure to upvote Peter's answer. $\newcommand{\GL}{\operatorname{GL}}$ $\newcommand{\ra}{\rightarrow}$ $\newcommand{\Z}{\mathbb{Z}}$


If $G$ is a group, then we say a group $S$ is a subquotient of $G$ if there are subgroups $N \subset H \subset G$ with $N$ normal in $H$ such that $H/N \cong S$.

Step 1: Let $G$ be a finite group. We claim that every simple subquotient of $G$ is a subquotient of some Jordan-Holder factor of $G$. By induction it suffices to show that if $N$ is a normal subgroup of $G$, then every simple subquotient of $G$ is a subquotient of either $N$ or $G/N$. Let $G'$ be a subgroup of $G$, let $N'$ be a normal subgroup of $G'$, and let $\pi: G \ra G/N$ be the quotient map. Then $G'$ is an extension of $\pi(G')$ by $G' \cap N$, so the Jordan-Holder factor $G'/N'$ of $G'$ must appear up to isomorphism as a Jordan-Holder factor of either $G' \cap N$ or of $\pi(G')$, and it follows that $G'/N'$ is a subquotient of either $N$ or $G/N$.

Step 2: Let $m \in \Z^+$. Let $n \in \Z^+$ have prime power factorization $n = p_1^{a_1} \cdots p_r^{a_r}$. Then $\GL_m(\Z/n\Z) \cong \prod_{i=1}^r \GL_m(\Z/p_i^{a_i} \Z)$. Further, for $1 \leq i \leq r$, the kernel of the natural map $\GL_m(\Z/p_i^{a_i} \Z) \ra \GL_m(\Z/p_i \Z)$ is a $p_i$-group. Thus every noncyclic simple subquotient of $\GL_m(\Z/n\Z)$ is a simple subquotient of $\GL_m(\Z/p_i \Z)$ for some $i$. Thus we assume henceforth that $n = p$ is prime.

Step 3: We apply the following theorem of Larsen-Pink: there is a positive integer $J'(m)$ such that every subgroup $\Gamma \subset \GL_m(\Z/p\Z)$ admits a subnormal series $\Gamma_3 \subset \Gamma_2 \subset \Gamma_1 \subset \Gamma$ such that:
$\bullet$ $[\Gamma:\Gamma_1] \leq J'(m)$;
$\bullet$ $\Gamma_1/\Gamma_2$ is a direct product of finite simple groups of Lie type;
$\bullet$ $\Gamma_2/\Gamma_3$ is commutative; and
$\bullet$ $\Gamma_3$ is a $p$-group.
In view of Step 1, the result follows immediately from this and the fact that for all $k \geq 9$, $A_k$ is not isomorphic to a finite simple group of Lie type.

$\endgroup$

Your Answer

By clicking “Post Your Answer”, you agree to our terms of service and acknowledge you have read our privacy policy.

Not the answer you're looking for? Browse other questions tagged or ask your own question.